LSAT and Law School Admissions Forum

Get expert LSAT preparation and law school admissions advice from PowerScore Test Preparation.

 Administrator
PowerScore Staff
  • PowerScore Staff
  • Posts: 8917
  • Joined: Feb 02, 2011
|
#35439
Complete Question Explanation

(See the complete passage discussion here: lsat/viewtopic.php?t=14276)

CR, Strengthen. The correct answer choice is (D)

This question asks which of the choices, if true of countries that shifted from graduated to flat tax
systems, would support the position of the second author, who supports a progressive tax system
over a flat tax system.

Answer choice (A): This choice would neither strengthen nor weaken either author’s argument
(though it is consistent with both).

Answer choice (B): If the tax codes have been simplified in the switch from a flat tax system to a
progressive tax system, that would support the argument of the first author, who asserts that a flat tax
is less complex than the systems operating in most developed countries (and that complex tax
codes provide the opportunity for high-income earners to avoid taxes).
Answer choice (C): The beliefs of high-income tax payers are not relevant to either argument, and
would not strengthen the argument presented in the second passage.

Answer choice (D): This is the correct answer choice. The second author argues that flat taxes are
not entirely sensible because they allow one to jump from paying no tax to paying the highest rate.
The author closes the passage with the point that if the working poor are not paying taxes, and the
wealthy are paying less taxes, under a flat tax the middle class will make up the difference. If, as this
choice provides, middle income earners that have switched to flat tax systems are now paying more,
that would lend support to the argument advanced at the end of the second passage.

Answer choice (E): First, the term “some” is very vague, referring to “at least one.” The fact that at
least one legislator has a preference would not strengthen or weaken either author’s argument.
 gintriag
  • Posts: 26
  • Joined: Sep 14, 2016
|
#29038
Hi,

In the first passage there is nothing about middle class, at least, explicitly written. So how is (D) the correct option?

Thanks for your help.
User avatar
 Jonathan Evans
PowerScore Staff
  • PowerScore Staff
  • Posts: 726
  • Joined: Jun 09, 2016
|
#29118
Hi, gintriag,

This question, a kind of logical reasoning question in disguise, asks you to strengthen the position of Passage B. Treat this question as you would the analogous logical reasoning problem. Identify the conclusion (or thesis/main point) of the passage.

Passage B contends that if a country adopts a flat tax rather than progressive taxation, the middle class will make up the difference in tax revenue. The fact that Passage A does not address the "middle class" is not crucial here as your primary concern is bolstering the thesis of Passage B. Since the question stem does ask you to support the "position of passage B over that of passage A" you do need to address the position of Passage A, to wit flat taxes result in a similar tax burden distribution. Now zero in on the difference in Passage B. Unlike Passage A, Passage B contends that the middle class would need to make up the difference in lost tax revenue. You need to find evidence to back up this claim.

As with many logical reasoning type questions in Reading Comprehension, when you treat these questions like their Logical Reasoning analogs, they actually can be quite accessible. Look for the answer choice that says the middle class pays more. Answer choice D does this, so that's it!

I hope this explanation helps.
User avatar
 ashpine17
  • Posts: 321
  • Joined: Apr 06, 2021
|
#92324
I got confused by the wording of the question. Is this a strengthen B only question or do I also have to weaken A?
User avatar
 ashpine17
  • Posts: 321
  • Joined: Apr 06, 2021
|
#92325
Why doesn't A work? Didn't passage B state that flat tax proposals are supposed to bring in the SAME amount of total tax revenue? As for A, I thought this choice did nothing for A
 Robert Carroll
PowerScore Staff
  • PowerScore Staff
  • Posts: 1787
  • Joined: Dec 06, 2013
|
#92906
ashpine,

Strengthening Passage B on this point should weaken Passage A, because their opinions on the effect of a flat tax are opposite.

That's why answer choice (A) doesn't work. Passage A would be completely fine with this happening. So it's not strengthening Passage B over Passage A.

Robert Carroll
User avatar
 ashpine17
  • Posts: 321
  • Joined: Apr 06, 2021
|
#92948
Oh, I didn't catch that at all. How are they expressing opposite viewpoints?
 Robert Carroll
PowerScore Staff
  • PowerScore Staff
  • Posts: 1787
  • Joined: Dec 06, 2013
|
#93015
ashpine,

Passage A is defending a flat tax and Passage B is attacking a flat tax.

Robert Carroll

Get the most out of your LSAT Prep Plus subscription.

Analyze and track your performance with our Testing and Analytics Package.